LSAT and Law School Admissions Forum

Get expert LSAT preparation and law school admissions advice from PowerScore Test Preparation.

 Administrator
PowerScore Staff
  • PowerScore Staff
  • Posts: 8917
  • Joined: Feb 02, 2011
|
#47407
Complete Question Explanation
(The complete setup for this game can be found here: lsat/viewtopic.php?t=17028)

The correct answer choice is (A)

If S testifies fifth, then T must testify sixth in order to conform to the first rule (U cannot testify sixth because of the actions of the second rule). Because only S, T, and U can testify first, and S and T already testify fifth and sixth, U must then testify first. Thus, answer choice (A) is correct.

Get the most out of your LSAT Prep Plus subscription.

Analyze and track your performance with our Testing and Analytics Package.